1 Background and Results

In order to put our results into context we first quickly review some related results for prescribed curvature problems on surfaces.

1.1 “Bubbling” Metrics on Closed Surfaces

Let (M,g0) be a closed Riemann surface with smooth background metric g0 and Euler characteristic χM < 0. Also let \(f_{0}\colon M\to \mathbb {R}\) be a smooth, non-constant function with \(\max \limits _{p\in M}f_{0}(p)=0\), and for any \(\lambda \in \mathbb {R}\) let fλ = f0 + λ. Then, if all maximum points p0 of f0 where f0(p0) = 0 are non-degenerate, it was shown by this author jointly with Borer and Galimberti [1] that for any sufficiently small λ > 0 there exists a conformal metric \(g_{\lambda }=e^{2u_{\lambda }}g_{0}\) on M of prescribed Gauss curvature fλ, which, as λ 0 suitably exhibits “bubbling” with finite volume and finite total curvature. Galimberti in [6] extended this result to the case when χM = 0. Finally, again in the case when χM = 0, in [11] a similar “bubbling” behavior in the limit regime when λ 0 was also observed for the corresponding prescribed curvature flow by this author.

The “bubbles” produced in this way are of the form \(g=e^{2u}g_{\mathbb {R}^{2}}\), where uC either solves the standard Liouville equation

$$ -{\Delta} u=e^{2u}\quad\text{ on } \mathbb{R}^{2} $$
(1)

or the equation

$$ -{\Delta} u=(1+(Ax,x))e^{2u}\quad\text{ on } \mathbb{R}^{2}, $$
(2)

where A is a negatively definite symmetric matrix given by \(A=\frac 12\text {Hess}_{p_{0}}f_{0}\) for some p0M with f0(p0) = 0. Moreover, they have finite volume and finite total curvature

$$ {\int}_{\mathbb{R}^{2}}e^{2u} dx<\infty,\quad {\int}_{\mathbb{R}^{2}}(1+(Ax,x))e^{2u} dx \in \mathbb{R}. $$
(3)

All solutions u with (3) to the Liouville equation (1) have been classified by Chen–Li [4] and they uniquely correspond to the solution obtained by pull-back of the round spherical metric on S2 under stereographic projection and its rescalings. Moreover, in [11] this author was able to rule out the existence of solutions uC to (2) satisfying (3). Thus, in n = 2 dimensions all “bubbles” resulting from blow-up are spherical.

1.2 “Bubbling” Metrics of Prescribed Q-Curvature

Similar results may be expected to hold true in the 4-dimensional case for the problem of prescribed Q-curvature. Let (M,g0) be a closed manifold of dimension n = 4 with smooth background metric g0 and suppose that the Paneitz operator P0 is non-negative with kernel consisting of constant functions only and with total Q-curvature \(k_{P_{0}}<0\). By a result of Chang–Yang [3], later generalized by Djadli–Malchiodi [5], we then may assume that (M,g0) has constant Q-curvature. For this case, and for smooth non-constant \(f_{0}\le 0=\max \limits _{M}f_{0}\), fλ = f0 + λ as above, Galimberti [7] established the existence of “bubbling” metrics as λ 0 analoguous to our results in [1], and Ngo–Zhang [9] obtained “bubbling” results for the case \(k_{P_{0}}=0\) which are the analogue of Galimberti’s [6] results for the 2-torus. Ngo–Zhang [9] also studied the corresponding prescribed Q-curvature flow and showed “bubbling” analoguous to the results in [11]

Similar to the 2-dimensional case, also in dimension n = 4 the “bubble” metrics \(g=e^{2u}g_{\mathbb {R}^{2}}\) are related to solutions uC either of the equation

$$ {\Delta}^{2} u=e^{4u}\quad\text{ on } \mathbb{R}^{4} $$
(4)

or of the equation

$$ {\Delta}^{2} u=K(x)e^{4u}\quad\text{ on } \mathbb{R}^{4}, $$
(5)

where K(x) results from the Taylor expansion of f0 around some p0M with f0(p0) = 0; moreover, the “bubble” metrics again have finite volume and finite total Q-curvature

$$ V_{0}={\int}_{\mathbb{R}^{4}}e^{4u} dx < \infty,\quad K_{0}={\int}_{\mathbb{R}^{4}}K(x)e^{4u} dx \in \mathbb{R}. $$
(6)

However, as shown by Chang–Chen [2], in contrast to the 2-dimensional case, in dimension n = 4 there is an abundance of non-spherical solutions to the equation (4) and also of solutions to the equation (5) for any given, smooth function K which is positive somewhere and satisfies |K(x)|≤ C|x|s for some s < 0, all satisfying (6). This prompted the question whether in n = 4 dimensions for certain functions f0 non-spherical blow-up might be possible.

Here we partially resolve this question and show that, in contrast to expectation, non-spherical blow-ups do not arise at points p0M with f0(p0) = 0, if dkf0(p0) = 0 for 1 ≤ k ≤ 3 while d4f0(p0)(x,x,x,x) < 0 for any x≠ 0. Any non-spherical blow-up solution then would be a solution of (5) for K(x) = 1 + A(x,x,x,x) with \(A=\frac {1}{24}d^{4}f_{0}(p_{0})\). A key observation is that, by an argument of Robert and this author [10], proof of Proposition 2.3, building on the work of Lin [8], for the above blow-up limits there additionally holds the condition that

$$ {\Delta} u(x)\to 0\quad \text{ as } |x|\to\infty. $$
(7)

With this extra information then we obtain the following result.

Theorem 1

Suppose that K(x) = 1 + A(x,x,x,x), where A is a negative definite and symmetric 4-linear map. Then there is no solution \(u\in C^{\infty }(\mathbb {R}^{4})\) of (5) with uC and satisfying (6) as well as (7).

Indeed, it is not too difficult to carry over the ideas from the proof of the analogous non-existence result Theorem 5.2 in our previous work [11] to the degenerate setting of Theorem 1. In contrast, it seems to be an interesting question whether the above result will also hold for the non-degenerate case when d2f0(p0) < 0 at all maximum points p0.

Notation

Throughout the letter C will denote a generic constant, occasionally numbered for clarity.

2 Proof of Theorem 1

We argue by contradiction. Let \(u \in C^{\infty }(\mathbb {R}^{4})\) be a solution of equation (5) with uC and satisfying Δu(x) → 0 as \(|x|\to \infty \). Moreover, assume that (6) holds true. Writing \(K(x)e^{4u}=(1+A(x,x,x,x))e^{4u}=:F\in L^{1}(\mathbb {R}^{4})\) for brevity, following Lin [8] we introduce

$$ v(x)=\frac{1}{8\pi^{2}}{\int}_{\mathbb{R}^{4}}(\log|x-y|-\log|y|)F(y) dy $$

formally solving the equation

$$ {\Delta}^{2}v=-F(y)\quad \hbox{ on }~\mathbb{R}^{4}. $$

Thus the function w = u + v satisfies Δ2w = 0. In fact, adapting the argument of [8] to our setting, from our assumptions (6) and (7) we see that v is indeed well-defined and that u can be represented in terms of v, as follows.

Lemma 1

The function v is well-defined and with a constant \(C \in \mathbb {R}\) there holds

$$ u(x)=C-v(x)=C-\frac{1}{8\pi^{2}}{\int}_{\mathbb{R}^{4}}(\log|x-y|-\log|y|)F(y) dy. $$
(8)

Proof

Since uC the function F is locally bounded and for any x the integral

$$ {\int}_{B_{1}(x)\cup B_{1}(0)}(\log|x-y|-\log|y|)F(y) dy $$

is well-defined. It thus suffices to bound \((\log |x-y|-\log |y|)\) for |y|≥ 1, |xy|≥ 1.

If |y|≥ 2|x| and hence |y|/2 ≤|xy|≤ 2|y| we can estimate

$$ |\log|x-y|-\log|y||=|\log(|x-y|/|y|)|\le\log2. $$

On the other hand, if |y| < 2|x|, for |y|≥ 1, |xy|≥ 1 we can bound

$$ \log|x-y|+\log|y|\le 2\log|x|+C, $$

and v(x) is well-defined for every \(x \in \mathbb {R}^{4}\).

Likewise we see that we may differentiate under the integral to obtain

$$ {\Delta} v(x)=\frac{1}{4\pi^{2}} {\int}_{\mathbb{R}^{4}} \frac{F(y)}{|x-y|^{2}} dy $$

and Δ2v = −F. Thus, letting h = Δu + Δv we find Δh = 0. In view of our assumption (7) that Δu(x) → 0 as \(|x|\to \infty \), and estimating

$$ \left|{\int}_{B_{1}(x_{0})}{\Delta} v dx\right| \le \frac{1}{4\pi^{2}}{\int}_{\mathbb{R}^{4}}{\int}_{B_{1}(x_{0})}\frac{|F(y)|}{|x-y|^{2}} dx dy \to 0\quad \text{ as }~|x_{0}|\to\infty, $$

from the mean value property of harmonic functions we then see that h ≡ 0, and w = u + v is harmonic. But estimating

$$ \begin{array}{@{}rcl@{}} &&{\int}_{B_{1}(x_{0})} {\int}_{B_{1}(x)}(\log|x-y|-\log|y|)F(y) dy dx\\ &&\qquad\le C{\int}_{B_{2}(x_{0})}{\int}_{B_{1}(x_{0})}\big|\log|x-y|\big||F(y)| dx dy + C\log|x_{0}|+C\\ &&\qquad\le C\log|x_{0}|+C, \end{array} $$

we obtain the bound

$$ {\int}_{B_{1}(x_{0})}v(x) dx\le C\log|x_{0}|+C $$

for |x0|≫ 1. Thus, since uC and again using the mean value property of harmonic functions, we find the bound \(w(x)=u(x)+v(x)\le C\log (2+|x|)\) for all \(x\in \mathbb {R}^{4}\). We conclude that wC for some \(C\in \mathbb {R}\), which gives the claim. □

We proceed, following the ideas of [11].

Lemma 2

There holds

$$ \frac{u(x)}{\log|x|}=\frac{C}{\log|x|}-\frac{1}{8\pi^{2}}{\int}_{\mathbb{R}^{4}} \frac{\log|x-y|-\log|y|}{\log|x|}F(y) dy~\to~ -\alpha $$
(9)

as \(|x|\to \infty \), where

$$ \alpha=\frac{1}{8\pi^{2}}{\int}_{\mathbb{R}^{4}}F(y) dy=\frac{K_{0}}{8\pi^{2}}. $$

Proof

Recalling that for |y|≥ 2|x| there holds

$$ \big|\log|x-y|-\log|y|\big|=\big|\log(|x-y|/|y|)\big|\le\log2, $$

we can write

$$ {\int}_{\mathbb{R}^{4}}\frac{\log|x-y|-\log|y|}{\log|x|}F(y) dy = {\int}_{|y|\le 2|x|}\frac{\log|x-y|-\log|y|}{\log|x|}F(y) dy+o(1) $$

with error o(1) → 0 as \(|x|\to \infty \). Moreover, again using that \(F\in L^{1}\cap L^{\infty }_{loc}(\mathbb {R}^{4})\) it is not hard to show that

$$ {\int}_{|y|\le 2|x|}\frac{\log|y|}{\log|x|}F(y) dy\to 0\quad (|x|\to\infty). $$

Also note that (8) and (7) with error o(1) → 0 as \(|x|\to \infty \) allow to bound

$$ \begin{array}{@{}rcl@{}} {\int}_{B_{1}(x)}\log|x-y|F(y) dy &\le& {\int}_{B_{1}(x))}\frac{-F(y)}{|x-y|^{2}}dy = -4\pi^{2}{\Delta} v(x)+o(1)\\ &=& 4\pi^{2}{\Delta} u(x)+o(1)\to 0\quad (|x|\to\infty). \end{array} $$

Finally, we observe that for any yB2|x|(0) ∖ B1(x) we have the uniform bound \(0\le \log |x-y|/\log |x|\le \log 3/\log |x|+1\) while for any fixed \(y\in \mathbb {R}^{4}\) as \(|x|\to \infty \) there holds \(\log |x-y|/\log |x|\to 1\). Again using that \(F\in L^{1}(\mathbb {R}^{4})\), from Lebesgue’s theorem on dominated convergence we then obtain the claim. □

From (9) and (6), for any μ > 4α − 4 we now can bound

$$ \begin{array}{@{}rcl@{}} {\int}_{\mathbb{R}^{4}\setminus B_{1}(0)}|x|^{-\mu} dx &\le& C{\int}_{\mathbb{R}^{4}}|x|^{4}e^{4u} dx+C\\ &\le& C{\int}_{\mathbb{R}^{4}}|A(x,x,x,x)|e^{4u} dx+C=C(V_{0}-K_{0})+C<\infty. \end{array} $$

It follows that α ≥ 2 and hence

$$ K_{0}\ge 16\pi^{2}>0. $$
(10)

Next, we multiply the terms in (5) with x ⋅∇u to find, on the one hand

$$ \begin{array}{@{}rcl@{}} {\Delta}^{2} ux \cdot \nabla u&=& \text{div}(\nabla{\Delta} u x \cdot \nabla u) - \nabla{\Delta} u\nabla u-\nabla{\Delta} u x \cdot \nabla^{2} u\\ &=& \text{div} (\nabla{\Delta} u x \cdot \nabla u - {\Delta} u\nabla u - {\Delta} u x \cdot \nabla^{2} u) + 2|{\Delta} u|^{2} + x \cdot \nabla\left( \frac{|{\Delta} u|^{2}}{2}\right)\\ &=&\text{div}\left( \nabla{\Delta} u x\cdot\nabla u-{\Delta} u\nabla u-{\Delta} u x\cdot\nabla^{2} u + x\frac{|{\Delta} u|^{2}}{2}\right), \end{array} $$
(11)

and on the other hand

$$ \begin{array}{@{}rcl@{}} K(x)e^{4u} x\cdot\nabla u&=&{\kern-1.5pt}(1+A(x,x,x,x))e^{4u} x\cdot\nabla u\\ &=&{\kern-1.5pt}\text{div}{\kern-1.5pt}\left( \frac{x}{4}(1{\kern-1.5pt}+{\kern-1.5pt}A(x,x,x,x))e^{4u}\right){}-{}({\kern-1.5pt}1{\kern-1.5pt}+{\kern-1.5pt}A(x,x,x,x))e^{4u} {\kern-1.5pt}-{\kern-1.5pt} A(x,x,x,x)e^{4u}\\ &=&\text{{\kern-1.5pt}div}{\kern-1.5pt}\left( \frac{x}{4}(1+A(x,x,x,x))e^{4u}\right) + e^{4u} - 2(1+A(x,x,x,x))e^{4u}. \end{array} $$

Integrating the latter equation, observing that by finiteness of \(\|F\|_{L^{1}}\) we have

$$ R{\int}_{\partial B_{R}(0)}(1+A(x,x,x,x))e^{4u}do\to 0 $$

as \(R\to \infty \) suitably, we then obtain

$$ {\int}_{B_{R}(0)}K(x)e^{4u} x\cdot\nabla u dx~\to~ V_{0}-2K_{0} $$
(12)

for suitable \(R\to \infty \).

Proposition 1

As \(R\to \infty \) there holds

$$ {\int}_{\partial B_{R}(0)}\frac{x}{R}\cdot\left( \nabla{\Delta} u x\cdot\nabla u-{\Delta} u\nabla u - {\Delta} u x\cdot\nabla^{2} u + x\frac{|{\Delta} u|^{2}}{2}\right)do\to-\frac{1}{16\pi^{2}}{K_{0}^{2}}. $$

Proof Proof of Theorem 1

Combining (10), (11), (12), and Proposition 1 we obtain

$$ 16\pi^{2}\le K_{0}\le V_{0}=2K_{0}-\frac{1}{16\pi^{2}}{K_{0}^{2}}=\frac{32\pi^{2}-K_{0}}{16\pi^{2}}K_{0}; $$

thus K0 = 16π2 = V0. But from the formula for K0 it follows that we must have K0 < V0, and a contradiction results. Thus, the proof of Theorem 1 is complete. □

It remains to show Proposition 1.

3 Proof of Proposition 1

We need the following estimate.

Lemma 3

There holds

$$ \lim_{|x|\to\infty}x\cdot\nabla u(x)=-\frac{K_{0}}{8\pi^{2}}; $$

moreover, we have

$$ |x||\nabla u(x)|\le C <\infty, $$

uniformly in \(x\in \mathbb {R}^{4}\).

Proof

Differentiating (8) we find

$$ x\cdot\nabla u(x) = -\frac{1}{8\pi^{2}}{\int}_{\mathbb{R}^{4}}\frac{x\cdot(x-y)}{|x-y|^{2}}F(y) dy = -\frac{K_{0}}{8\pi^{2}}+I(x), $$

with

$$ I(x):=-\frac{1}{8\pi^{2}}{\int}_{\mathbb{R}^{4}}\frac{y\cdot(x-y)}{|x-y|^{2}}F(y) dy. $$

Also consider the term

$$ II(x)=x^{\perp}\cdot\nabla u(x) = -\frac{1}{8\pi^{2}}{\int}_{\mathbb{R}^{4}}\frac{x^{\perp}\cdot(x-y)}{|x-y|^{2}}F(y) dy, $$

for an arbitrary rotation P : xx on \(\mathbb {R}^{4}\) such that xx = 0 for all \(x\in \mathbb {R}^{4}\).

Given any ε > 0 we claim that for sufficiently large R > 0 we can estimate the error terms

$$ \sup_{|x|=R} \left( |I(x)|+|II(x)|\right) \le C\varepsilon $$

with a constant C > 0 independent of R.

Let ε > 0 be given. There exists R0 > 1 such that

$$ {\int}_{\mathbb{R}^{4} \setminus B_{R_{0}}(0)}|F(y)| dy<\varepsilon. $$

Let |x| = 2R ≥ 2R0. Observing that |y|≤|xy| + |x| gives

$$ \frac{|y|}{|x-y|}\le 3\quad \hbox{ for }y\notin B_{R}(x), $$

we can bound

$$ \begin{array}{@{}rcl@{}} &&\left|8\pi^{2} I(x)+{\int}_{B_{R}(x)}\frac{y\cdot(x-y)}{|x-y|^{2}}F(y) dy\right| \\ &&\qquad \le{\int}_{\mathbb{R}^{4}\setminus (B_{R}(x)\cup B_{R_{0}}(0))}\frac{|y|}{|x-y|}|F(y)| dy + \frac{R_{0}}{2R-R_{0}}{\int}_{B_{R_{0}}(0)}|F(y)| dy\\ &&\qquad\le C\varepsilon+\frac{CR_{0}}{2R-R_{0}}\le C\varepsilon \end{array} $$

if RR0 is sufficiently large. Moreover, for RR0, |x| = 2R we have

$$ \left|{\int}_{B_{R}(x)}\frac{y\cdot(x-y)}{|x-y|^{2}}F(y) dy - {\int}_{B_{R}(x)}\frac{x\cdot(x-y)}{|x-y|^{2}}F(y) dy\right| \le {\int}_{B_{R}(x)}|F(y)| dy\le\varepsilon. $$

Similarly, observing that

$$ 8\pi^{2} II(x)=-{\int}_{\mathbb{R}^{4}}\frac{x^{\perp}\cdot(x-y)}{|x-y|^{2}}F(y) dy = -{\int}_{\mathbb{R}^{4}}\frac{y^{\perp}\cdot(x-y)}{|x-y|^{2}}F(y) dy, $$

we can bound

$$ \begin{array}{@{}rcl@{}} &&\left|8\pi^{2}II(x)+{\int}_{B_{R}(x)}\frac{y^{\perp}\cdot(x-y)}{|x-y|^{2}}F(y) dy\right|\\ &&\qquad\le{\int}_{\mathbb{R}^{4}\setminus (B_{R}(x)\cup B_{R_{0}}(0))}\frac{|y|}{|x-y|}|F(y)| dy + \frac{R_{0}}{2R-R_{0}}{\int}_{B_{R_{0}}(0)}|F(y)| dy\\ &&\qquad\le C\varepsilon+\frac{CR_{0}}{2R-R_{0}}\le C\varepsilon \end{array} $$

if RR0 is sufficiently large, and we have

$$ {\int}_{B_{R}(x)}\frac{y^{\perp}\cdot(x-y)}{|x-y|^{2}}F(y) dy = {\int}_{B_{R}(x)}\frac{x^{\perp}\cdot(x-y)}{|x-y|^{2}}F(y) dy. $$

Thus, it suffices to bound the integrals

$$ {\int}_{B_{R}(x)}\frac{x\cdot(x-y)}{|x-y|^{2}}F(y) dy \quad \hbox{ and }\quad {\int}_{B_{R}(x)}\frac{x^{\perp}\cdot(x-y)}{|x-y|^{2}}F(y) dy, $$

respectively. Changing coordinates to z = xy we obtain

$$ \begin{array}{@{}rcl@{}} {\int}_{B_{R}(x)}\frac{x\cdot(x-y)}{|x-y|^{2}}F(y) dy &=& {\int}_{B_{R}(0)}\frac{x\cdot z}{|z|^{2}}F(x-z) dz\\ &=&\frac12{\int}_{B_{R}(0)}\frac{x\cdot z}{|z|^{2}}(F(x-z)-F(x+z)) dz, \end{array} $$

where we observe that

$$ {\int}_{B_{R}(0)}\frac{x\cdot z}{|z|^{2}}(F(x-z)+F(x+z)) dz=0 $$

by symmetry. Expanding

$$ \begin{array}{@{}rcl@{}} A(x\pm z,x\pm z,x\pm z,x\pm z) &=& A(x,x,x,x)\pm 4A(x,x,x,z)\\ &&+6A(x,x,z,z)\pm 4A(x,z,z,z)+A(z,z,z,z) \end{array} $$

we then have

$$ \begin{array}{@{}rcl@{}} F(x-z)-F(x+z) &=& \left( 1+A(x-z,x-z,x-z,x-z)\right)e^{4u(x-z)}\\ &&-\left( 1+A(x+z,x+z,x+z,x+z)\right)e^{4u(x+z)}\\ &=&\left( (1+A(x,x,x,x)+6A(x,x,z,z)+A(z,z,z,z)\right)\left( e^{4u(x-z)}-e^{4u(x+z)}\right)\\ &&-4\left( A(x,x,x,z)+A(x,z,z,z)\right)\left( e^{4u(x-z)}+e^{4u(x+z)}\right). \end{array} $$

Note that we can bound

$$ \begin{array}{@{}rcl@{}} &&{\int}_{B_{R}(0)}\frac{|x\cdot z|}{|z|^{2}}\left( |A(x,x,x,z)+A(x,z,z,z)|\right) \left( e^{4u(x-z)}+e^{4u(x+z)}\right) dz\\ &&\quad\le C{\int}_{B_{R}(0)}|x|^{4}\left( e^{4u(x-z)}+e^{4u(x+z)}\right) dz \le C{\int}_{B_{R}(x)}|F(y)| dy\le C\varepsilon. \end{array} $$

Similarly, we have

$$ \begin{array}{@{}rcl@{}} &&{\int}_{B_{R}(0)}\frac{|x\cdot z|}{|z|^{2}}\big|6A(x,x,z,z)+A(z,z,z,z)\big|\left( e^{4u(x-z)}+e^{4u(x+z)}\right) dz\\ &&\quad\le C{\int}_{B_{R}(0)}|x|^{4}\left( e^{4u(x-z)}+e^{4u(x+z)}\right) dz \le C{\int}_{B_{R}(x)}|F(y)| dy\le C\varepsilon. \end{array} $$

By Lemma 2 and (10) we also can bound e4u(x±z)R− 4μ for any μ < 2 when |x| = 2R ≥ 2|z| with sufficiently large R ≫ 1. Choosing μ = 3/2 we find

$$ {\int}_{B_{R}(0)}\frac{|x\cdot z|}{|z|^{2}}\left( e^{4u(x-z)}+e^{4u(x+z)}\right) dz \le CR^{1-4\mu}{\int}_{B_{R}(0)}\frac{dz}{|z|}\le CR^{4-4\mu}\to 0 $$

as \(|x|=2R\to \infty \).

Thus we obtain

$$ \left|8\pi^{2} I(x)+\frac12{\int}_{B_{R}(0)}\frac{x\cdot z}{|z|^{2}}A(x,x,x,x) \left( e^{4u(x-z)}-e^{4u(x+z)}\right) dz\right| \le C\varepsilon, $$

and by the same type of reasoning also that

$$ \left|8\pi^{2} II(x)+\frac12{\int}_{B_{R}(0)}\frac{x^{\perp}\cdot z}{|z|^{2}}A(x,x,x,x) \left( e^{4u(x-z)}-e^{4u(x+z)}\right) dz\right| \le C\varepsilon $$

if RR0 is sufficiently large.

But letting i,j,kS3 be the usual imaginary quaternions with i2 = j2 = k2 = ijk = − 1, and integrating for each z0 with |z0| = r < R from z0 to − z0 along a semi-circle S(z0) of radius r, parametrized by 𝜃 ∈ [0,π], with tangent vector iz at any z = z(𝜃) ∈ S(z0), we have

$$ \begin{array}{@{}rcl@{}} \left|e^{4u(x-z_{0})}-e^{4u(x+z_{0})}\right| &\le& \left|{\int}_{0}^{\pi}4iz\cdot\nabla u(x+z)e^{4u(x+z)} d\theta\right|\\ &\le& C\sup_{|z|=r}|\nabla u(x+z)|{\int}_{S(z_{0})}e^{4u(x+z)} ds, \end{array} $$

where ds denotes arc-length. Hence for any |x| = 2R and sufficiently large RR0 there results

$$ \begin{array}{@{}rcl@{}} |I(x)|+\sup_{P\in O(4)}|II(x)| &\le& C\sup_{|z|\le R}|\nabla u(x+z)|{\int}_{B_{R}(0)}|x|^{5}e^{4u(x+z)} dz+C\varepsilon\\ &\le& C\sup_{y\in B_{R}(x)}|y||\nabla w(y)|{\int}_{B_{R}(x)}|F(y)| dy+C\varepsilon\\ &\le& C\varepsilon\sup_{y\in B_{R}(x)}|y||\nabla w(y)|+C\varepsilon. \end{array} $$
(13)

But for any μ < 2 for any sufficiently large |x| = 2R ≥ 2R1 = 2R1(μ) ≥ 2R0 ≥ 2 and any yBR(x) we can bound |F(y)|≤ C|y|4e4u(y)CR4 − 4μ to obtain

$$ \begin{array}{@{}rcl@{}} |\nabla u(x)|&=&\left|{\int}_{\mathbb{R}^{4}}\frac{x-y}{|x-y|^{2}}F(y) dy\right| \le {\int}_{\mathbb{R}^{4}\setminus B_{R}(x)}\frac{|F(y)|}{|x-y|} dy +{\int}_{B_{R}(x)}\frac{|F(y)|}{|x-y|} dy\\ &\le& CR^{-1}+CR^{7-4\mu}\le CR^{7-4\mu}. \end{array} $$

Thus, for any 0 < ν < 1 the number \(\sup _{y\in \mathbb {R}^{4}}|y|^{\nu }|\nabla u(y)|\) is attained.

We claim that this also implies that

$$ \sup_{y\in\mathbb{R}^{4}}|y||\nabla u(y)|<\infty. $$

Otherwise for ν 1 there exist points \(x_{\nu }\in \mathbb {R}^{4}\) with \(|x_{\nu }|=:2R_{\nu }\to \infty \) such that

$$ \gamma_{\nu}:=|x_{\nu}|^{\nu}|\nabla u(x_{\nu})|=\sup_{x\in\mathbb{R}^{4}}|x|^{\nu}|\nabla u(x)|\to\infty~\hbox{ as }~\nu\uparrow 1. $$

But from the definitions of I and II and estimate (13) above, with a constant C2 > 0 independent of ν we have

$$ \begin{array}{@{}rcl@{}} \gamma_{\nu} &=& |x_{\nu}|^{\nu}|\nabla u(x_{\nu})| \le \left( \frac{K_{0}}{8\pi^{2}}+|I(x_{\nu})|+\sup_{P\in O(4)}|II(x_{\nu})|\right)|x_{\nu}|^{\nu-1}\\ &\le& \left( \frac{K_{0}}{8\pi^{2}}+C\varepsilon\sup_{y\in B_{R_{\nu}}(x_{\nu})}|y||\nabla u(y)| + C\varepsilon\right)|x_{\nu}|^{\nu-1}\\ &\le& \left( \frac{K_{0}}{8\pi^{2}}+C_{2}\varepsilon\right)|x_{\nu}|^{\nu-1} + C_{2}\varepsilon\sup_{|y|\ge R_{\nu}}|y|^{\nu}|\nabla u(y)| \end{array} $$

where we may let ε → 0 as \(|x_{\nu }|=2R_{\nu }\to \infty \). In particular, for sufficiently large ν < 1 (and hence sufficiently large Rν > 1) we have C2ε < 1/2 and we can bound

$$ \gamma_{\nu} \le \left( \frac{K_{0}}{8\pi^{2}}+C_{2}\varepsilon\right)|x_{\nu}|^{\nu-1} + C_{2}\varepsilon\sup_{|y|\ge R_{\nu}}|y|^{\nu}|\nabla u(y)| \le \left( \frac{K_{0}}{8\pi^{2}}+\frac12\right) +\frac{\gamma_{\nu}}{2}. $$

Thus for any sufficiently large ν < 1 there holds

$$ \gamma_{\nu}\le\frac{K_{0}}{4\pi^{2}}+1. $$

It follows that for every \(x\in \mathbb {R}^{4}\) we can bound

$$ |x||\nabla u(x)|\le\limsup_{\nu\uparrow 1}|x|^{\nu}|\nabla u(x)| \le \limsup_{\nu\uparrow 1}\gamma_{\nu}\le\frac{K_{0}}{4\pi^{2}}+1. $$

Together with (13) this concludes the proof. □

As an immediate consequence of Lemma 3 we obtain the following Harnack-type estimate.

Lemma 4

There exists a constant C > 0 such that for any R > 1, any \(x\in \mathbb {R}^{4}\) with |x| = 2R there holds

$$ \sup_{y\in B_{R}(x)}|F(y)|\le C\inf_{y\in B_{R}(x)}|F(y)|. $$

Proof

For any yBR(x) and any 0 ≤ t ≤ 1 we can bound

$$ \left|\frac{d}{dt}F(x+t(y-x))\right| = |(y-x)\cdot\nabla F(x+t(y-x))|\le R|\nabla F(x+t(y-x))|. $$

Moreover, for any zBR(x) in view of the bound |z|≥|x|− R = R there holds R|∇A(z)|≤ 4|A(z)|, where we denote A(z,z,z,z) as A(z) for brevity. Thus, by Lemma 3 for any zBR(x) we have

$$ R|\nabla F(z)| \le 4\left( 1+|z||\nabla u(z)|\right)|F(z)|\le C|F(z)|. $$

It follows that

$$ \left|\frac{d}{dt}F(x+t(y-x))\right| \le C|F(x+t(y-x))|,\quad 0\le t\le 1, $$

and

$$ |F(y)/F(x)|+|F(x)/F(y)|\le C. $$

The claim follows. □

Proof Proof of Proposition 1

Differentiating (8) further, for xBR(0) we also find the representations

$$ {\Delta} u(x) = -\frac{1}{4\pi^{2}}{\int}_{\mathbb{R}^{4}}\frac{F(y)}{|x-y|^{2}} dy=-\frac{1}{4\pi^{2}R^{2}}K_{0}+III(x) $$

with

$$ III(x):=\frac{1}{4\pi^{2}} {\int}_{\mathbb{R}^{4}}\left( \frac{1}{|x|^{2}}-\frac{1}{|x-y|^{2}}\right)F(y) dy, $$

as well as

$$ \left( x\cdot\nabla^{2} u(x),x\right) = -\frac{1}{8\pi^{2}}{\int}_{\mathbb{R}^{4}}\frac{|x|^{2}|x-y|^{2}-2|x\cdot(x-y)|^{2}}{|x-y|^{4}}F(y) dy, $$

and

$$ x\cdot\nabla{\Delta} u(x)=\frac{1}{2\pi^{2}}{\int}_{\mathbb{R}^{4}}\frac{x\cdot(x-y)F(y)}{|x-y|^{4}} dy = \frac{1}{2\pi^{2}R^{2}}K_{0}+IV(x), $$

where

$$ IV(x):=\frac{1}{2\pi^{2}} {\int}_{\mathbb{R}^{4}}\left( \frac{x\cdot(x-y)}{|x-y|^{4}}-\frac{1}{|x|^{2}}\right)F(y) dy. $$

Note that we can combine the terms

$$ x\cdot\left( x\frac{|{\Delta} u|^{2}}{2}-{\Delta} u\nabla u-{\Delta} u x\cdot\nabla^{2} u\right)={\Delta} u\left( |x|^{2}\frac{\Delta u}{2}-x\cdot\nabla u-(x\cdot\nabla^{2} u(x),x)\right) $$

with

$$ \begin{array}{@{}rcl@{}} |x|^{2}\frac{\Delta u}{2}-x\cdot\nabla u-(x\cdot\nabla^{2} u(x),x) &=& \frac{1}{8\pi^{2}}{\int}_{\mathbb{R}^{4}}\left( \frac{x\cdot(x-y)}{|x-y|^{2}} -\frac{2|x\cdot(x-y)|^{2}}{|x-y|^{4}}\right)F(y) dy\\ &=&-\frac{1}{8\pi^{2}}K_{0}-I(x)+V(x), \end{array} $$

where I as in the proof of Lemma 3 satisfies I(x) → 0 as \(|x|\to \infty \), and where

$$ V(x):=\frac{1}{4\pi^{2}} {\int}_{\mathbb{R}^{4}}\left( 1-\frac{|x\cdot(x-y)|^{2}}{|x-y|^{4}}\right)F(y) dy. $$

Replacing R by 2R for later convenience, we now claim that we can estimate the error terms

$$ \sup_{|x|=2R}\left( |x|^{2}|III(x)|+|x|^{2}|IV(x)|+|V(x)|\right)\to 0 $$
(14)

as \(R\to \infty \). From this the asserted convergence follows.

Fix ε > 0 and let R0 > 1 such that

$$ {\int}_{\mathbb{R}^{4}\setminus B_{R_{0}}(0)}|F(y)| dy<\varepsilon $$
(15)

as in the proof of Lemma 3. For |x| = 2R, |y|≤ R0, for sufficiently large RR0 we can bound

$$ \left|\frac{|x|^{2}-|x-y|^{2}}{|x-y|^{2}}\right|=\frac{\big|2x\cdot y-|y|^{2}\big|}{|x-y|^{2}} \le \frac{2RR_{0}+{R_{0}^{2}}}{(2R-R_{0})^{2}}\le\frac{3R_{0}}{2R-R_{0}}<\varepsilon. $$

Similarly, we have

$$ \left|\frac{|x|^{2}x\cdot(x-y)-|x-y|^{4}}{|x-y|^{4}}\right| \le C\frac{R^{3}R_{0}}{(2R-R_{0})^{4}} \le \frac{CR_{0}}{2R-R_{0}}<\varepsilon $$

and

$$ \left|\frac{|x\cdot(x-y)|^{2}-|x-y|^{4}}{|x-y|^{4}}\right| \le C\frac{R^{3}R_{0}}{(2R-R_{0})^{4}}<\varepsilon $$

for sufficiently large RR0. Moreover, we observe that for yBR(x) the terms on the left in the above three formulas are uniformly bounded. Thus for any x with |x| = 2R by (15) the contributions on \(\mathbb {R}^{4}\setminus B_{R}(x)\) to the error terms (14) are bounded by Cε.

Finally, for yBR(x) we can estimate

$$ \left|\frac{|x|^{2}-|x-y|^{2}}{|x-y|^{2}}\right| \le \frac{|x|^{2}}{|x-y|^{2}}+1\le 2\frac{|x|^{2}}{|x-y|^{2}} \le 2\frac{|x|^{3}}{|x-y|^{3}}, $$

as well as

$$ \left|\frac{|x|^{2}x\cdot(x-y)-|x-y|^{4}}{|x-y|^{4}}\right| \le \frac{|x|^{3}}{|x-y|^{3}}+1\le 2\frac{|x|^{3}}{|x-y|^{3}}, $$

and

$$ \left|\frac{|x\cdot(x-y)|^{2}-|x-y|^{4}}{|x-y|^{4}}\right| \le \frac{|x|^{2}}{|x-y|^{2}}+1\le 2\frac{|x|^{2}}{|x-y|^{2}}\le 2\frac{|x|^{3}}{|x-y|^{3}}, $$

to see that

$$ |x|^{2}|III(x)|+|x|^{2}|IV(x)|+|V(x)| \le C{\int}_{B_{R}(x)}\frac{|x|^{3}}{|x-y|^{3}}|F(y)| dy+C\varepsilon. $$

But by Lemma 4 for |x| = 2R we can bound

$$ \begin{array}{@{}rcl@{}} {\int}_{B_{R}(x)}\frac{|x|^{3}}{|x-y|^{3}}|F(y)| dy &\le& \sup_{y\in B_{R}(x)}|F(y)|{\int}_{B_{R}(x)}\frac{|x|^{3}}{|x-y|^{3}} dy\\ &\le& CR^{4}\sup_{y\in B_{R}(x)}|F(y)|\le CR^{4}\inf_{y\in B_{R}(x)}|F(y)|\\ &\le& C{\int}_{B_{R}(x)}|F(y)| dy\le C\varepsilon, \end{array} $$

and

$$ |x|^{2}|III(x)|+|x|^{2}|IV(x)|+|V(x)|\le C\varepsilon, $$

as claimed. □